LSAT and Law School Admissions Forum

Get expert LSAT preparation and law school admissions advice from PowerScore Test Preparation.

User avatar
 Dave Killoran
PowerScore Staff
  • PowerScore Staff
  • Posts: 5853
  • Joined: Mar 25, 2011
|
#82686
Complete Question Explanation
(The complete setup for this game can be found here: lsat/viewtopic.php?t=867)

The correct answer choice is (A).

Answer choices (A) is possible when the rounds are odd-even: when 3 plays 2, J first defeats S and holds position, and then when 2 plays 1, J defeats R and rises to position 1. Thus, answer choice (A) is possible and correct.

Answer choices (B) and (C) are the same type of answer: a player at one of the “ends” trying to win or lose two matches. This is not possible due to the way the matches leave a player out each time. As the position 5 player, if L plays in the first round, it must be an odd round. If L loses the first match, then L would not play in the second round,which would be even. Similarly, as the position 1 player, if R plays in the first round, it must be an even round. If R wins the first match, then R would not play in the second round,which would be odd.

Answer choice (D) is incorrect because J and L are too far apart for L’s only match to be against J.

Answer choice (E) is incorrect since there cannot be consecutive identical matches as part of the pattern of the game.
 salsaden
  • Posts: 8
  • Joined: May 02, 2012
|
#22228
Hi I have been working through this problem and I have been having difficulty understanding and setting up the problem. IT seems like there are more possibilities than I could possibly diagram. For example question 21- asks what could be rue after exactly two rounds of matches have been played- a match could begin with even or odd and then you have to diagram all of the possibilities for whether each pair of teams wins or loses. I tried to do a general diagram with an odd position round and an even position round but even then I could only narrow the answer choices down to A and B. I feel I am missing a key inference about this game?
User avatar
 Dave Killoran
PowerScore Staff
  • PowerScore Staff
  • Posts: 5853
  • Joined: Mar 25, 2011
|
#22229
Hey Salsaden,

Thanks for the question. Let's look at it a bit more closely.

With this question, I'd say that trying to show all the possibilities is not the right approach. Doing so somewhat plays into the hands of the test makers because you are guaranteed to lose a lot of time trying to show the many different things that could occur in a question this broad. So, then, what's the right approach? Well, first consider the nature of the game. It's a Pattern game, and those often have a large number of solutions. This is why there are often many Local questions in Pattern games. Next, consider the nature of this question. In this case, this is a Global, Could Be True question. when these questions appear in Pattern games, they often reflect some truth that underlies the Pattern, and by examining the nature of each answer choice you can often eliminate a few. Then, if multiple answer remain, you can try a hypothetical or two.

Here's an explanation for each answer. Note how the four wrong answers can all be eliminated just from an analysis of the pattern and the initial lineup of variables (explanations drawn from the PowerScore LSAT Logic Games Setups Encyclopedia, Volume 1 http://www.powerscore.com/lsat/content_ ... s_lgse.cfm):

Answer choices (A) is possible when the rounds are odd-even: when 3 plays 2, J first defeats S and holds position, and then when 2 plays 1, J defeats R and rises to position 1. Thus, answer choice (A) is possible and correct.

Answer choices (B) and (C) are the same type of answer: a player at one of the “ends” trying to win or lose two matches. This is not possible due to the way the matches leave a player out each time. As the position 5 player, if L plays in the first round, it must be an odd round. If L loses the first match, then L would not play in the second round,which would be even. Similarly, as the position 1 player, if R plays in the first round, it must be an even round. If R wins the first match, then R would not play in the second round,which would be odd.

Answer choice (D) is incorrect because J and L are too far apart for L’s only match to be against J.

Answer choice (E) is incorrect since there cannot be consecutive identical matches as part of the pattern of the game.

Please let me know if that helps. Thanks!
 salsaden
  • Posts: 8
  • Joined: May 02, 2012
|
#22230
Thank you, this helps a lot, I ran into trouble with this one bc I did not identify it as a pattern game until afterwards but it is easier to deal with once I understood the pattern better.
 ruffiebesq
  • Posts: 1
  • Joined: May 15, 2013
|
#9885
Thanks so much. Question 21. on the 1995 LSAT HuH? I had a method of diaplaying my possibilities before I began the game and this one has me stuck!!!! The switching positions and the odd and even positions confuse me!!!!!.
Which one of the following could be true after exactly
two rounds of matches have been played?
(A) J has won two matches.
(B) L has lost two matches.
(C) R has won two matches.
(D) L’s only match was played against J.
(E) M played against S in two matches
 Jason Schultz
PowerScore Staff
  • PowerScore Staff
  • Posts: 49
  • Joined: Jun 13, 2013
|
#9891
Hello, welcome, and thanks for the question!

As you know from the first rule, the matches always alternate between even and odd, but note that the rule does not specify which comes first. Indeed, this is not stated anywhere so you must be aware that the league can begin with an even round or an odd round. So after two rounds (the local rule here), the league has either gone odd-even or even-odd. Since the stem is a Could Be True, an answer choice which appears in either scenario is the correct one.

An even round is 1v2 and 3v4 with the 5 seed resting, and odd round is 2v3 and 4v5 with the 1 seed resting.

I diagrammed those as Even: (12)(34)5
Odd: 1(23)(45)
Answer choices A, B, and C are all about winning or losing two consecutive matches. But notice that since one player is resting in either an even or odd round, not every player even gets to play two consecutive matches. R begins at the #1 seed, and if the game begins in an Even round they cannot even play two games. If it begins in an Odd round, and R wins as required by (C), then in the following Even round they would rest, as they have retained the #1 seed. The same analysis eliminates L, if you swap "win" for "lose".

J, however, will always play two matches. If the game begins with an Even, they would be defending their spot against S, and then a win would have them challenging R for the #1 seed in the following round. Since they can appear in both matches, and win them, (A) is the correct answer.

Get the most out of your LSAT Prep Plus subscription.

Analyze and track your performance with our Testing and Analytics Package.